A resistor and a capacitor are connected in series across an ideal battery having a constant voltage across its terminals. Long after contact is made with the battery (a) the voltage across the capacitor is A) equal to the battery's terminal voltage. B) less than the battery's terminal voltage, but greater than zero. C) zero. (b) the voltage across the resistor is A) equal to the battery's terminal voltage. B) less than the battery's terminal voltage, but greater than zero. C) zero.

Answers

Answer 1

Answer:

A) equal to the battery's terminal voltage.

Explanation:

When the capacitor is fully charged after long hours of charging , its  potential becomes equal to the emf of the battery and its polarity is opposite to that of battery . Hence net emf becomes equal . The capacitor itself becomes a battery which is connected in the circuit with opposite polarity . This results in the net emf and  current becoming zero . There is no charging current when the capacitor is fully charged .


Related Questions

1. You are playing with a jump rope that is tied at both ends. You untie one end, hold it taut and wiggle the end up and down sinusoidally with frequency 2.00Hz and amplitude 0.075m. At time t=0, the end has a maximum positive displacement and is instantaneously at rest. Assume no wave bounces back from the far end to change the pattern. What is the equation for the displacement of the wave? What is the displacement at a point 3.00m from the end .

Answers

Answer:

[tex]f(x=3.00m)=0.075mcos(\frac{2\pi(2.00Hz)}{v}(3.00m))[/tex]

Explanation:

To find the equation of the wave you use the general equation for a wave, given by:

[tex]f(x)=Acos(k x-\omega t)[/tex]

A: amplitude of the wave = 0.075m

k: wave number

you select a cosine function because for x=0 and t= 0 you get a maximum displacement.

To find the displacement of the wave for x=0 you can consider that the form of the wave is independent of time t.

Then, you calculate k:

[tex]k=\frac{\omega}{v}=\frac{2\pi f}{v}[/tex]

Thus, you need the value of the speed of the wave (you only have the frequency f), in order to calculate f(x), for x=3.00m:

[tex]f(x=3.00m)=0.075mcos(\frac{2\pi(2.00Hz)}{v}(3.00m))[/tex]

which one of the following statements is true? A.in an elastic collision,only momentum is conserved B. in any collision,both momentum & kinetic energy are conserved C.in an inelastic collision,both momentum & kinetic energy are conserved D.in an elastic collision,only kinetic energy is conserved ​

Answers

Answer:

option C is correct

................

Answer:

C- in an inelastic collision, both momentum & kinetic energy are conserved

Explanation:

Took the test

Modified Newtonian dynamics(MoND)proposes that, for small accelerations, Newton’s second law, F = ma, approaches the form F = ma2/a0, where a0 is a constant.

(a) (10 points) Show how such a modified version of Newton’s second law can lead to flat rotation curves, without the need for dark matter.
(b) (10 points) Alternatively, propose a new law of gravitation to replace F = GMm/r2 at distances greater than some characteristic scale r0 so that again, you can explain the observed flat rotation curved of galaxies without dark matter.

Answers

Answer:

Explanation:

The two pictures attached here shows the solution to the two questions from the problem. thank you and I hope it helps you

Dual Nature of Light
Assignment
Active
Explaining the Nature of Light
Why do scientists believe that light is made of streams of
particles?

Answers

scientists believe that light is made of streams of particles because only the presence of photons can explain phenomena observed during experiments on the photoelectric effect.

What happens if you move a magnet near a could of wire

Answers

Answer:

The wire would stick to the magnet????????????????????????

Explanation:

Why does current flow in a coil when a magnet is pushed in and out of the coil ?

Answers

Answer:

So the induced current opposes the motion that induced it (from Lenz's Law). When we pull the magnet out, the left hand end of the coil becomes a south pole (to try and hold the magnet back). Therefore the induced current must be flowing clockwise.

hope this helps u...

In order to get going fast, eagles will use a technique called stooping, in which they dive nearly straight down and tuck in their wings to reduce their surface area. While stooping, a 6- kg golden eagle can reach speeds of up to 53 m/s . While golden eagles are not very vocal, they sometimes make a weak, high-pitched sound. Suppose that while traveling at maximum speed, a golden eagle heads directly towards a pigeon while emitting a sound at 1.1 kHz. The emitted sound has a sound intensity level of 30 dB when heard at a distance of 5 m .A) Model this stooping golden eagle as an object moving at terminal velocity. The eagle’s drag coefficient is 0.5 and the density of air is 1.2 kg/m 3 . What is the effective cross-sectional area of the eagle’s body while stooping?B) What is the doppler-shifted frequency that the pigeon will hear coming from the eagle?C) Consider the moment when the pigeon is 5 m away from the eagle. At the pigeon’s position, what is the intensity (in W/m^2 ) of the sound the eagle makes?D) The golden eagle slams into the 250- g pigeon, which is initially moving at 10 m/s in the opposite direction (toward the eagle). The eagle grabs the pigeon in its talons, and they move off together in a perfectly inelastic collision. How fast do they move after the collision?

Answers

Answer:

Check the explanation

Explanation:

Part A

F = CA

this drag force balances the weight = 6X 9.8

so

6X9.8 = 0.5 X A X0.5 X 1.2 X 532

A= 0.069 m2

Part B

here the sorce is moving and the observer is at rest

so f= f(- 1 - 1

f = 1.1X10 343 343 – 53

f' = 1.3 KHz

Part C:

given the intensity = 30 dB

we know that I dB = 10 log (I(W/m2))

so we get I (W/m2) = 1000

Part D : The catch

Given that U1 = 53 M1 = 6 kg

U2 =-10 M2=0.25

V1=V2

now conserving momentum

6 X 53 -0.25 X10 =(6+0.25)V

V= 50.48 m/sec


1) Calculate the equivalent resistance of 2 resistors wired in parallel. The value of the resistors are
4 ohms and 8 ohms.

Answers

Explanation:

[tex]r = \: ( { \frac{4 \times 8}{4 + 8} } \\ r= \: ohm[/tex]2.67

Answer:2 2/3 ohms

Explanation:

equivalent resistance=R

r1=4 ohms

r2=8 ohms

For parallel connection

1/R=1/(r1) + 1/(r2)

1/R=1/4 + 1/8

1/R=(2x1+1x1)/8

1/R=(2+1)/8

1/R=3/8

Cross multiply

1x8=3xR

8=3R

Divide both sides by 3

8/3=3R/3

8/3=R

R=8/3 or 2 2/3 ohms

Air is matter which backs best support the statement

Answers

Answer: A. Balloons can be filled with air.

C. Air has mass.

Explanation:

Learn more https://brainly.com/question/3238218

Balloons are able to be filled with air and air has mass.

Find the frequency of the 4th harmonic waves on a violin string that is 48.0cm long with a mass of 0.300 grams
and is under a tension of 4.00N. ​

Answers

Answer:

The frequency of the 4th harmonic of the string is 481.13 Hz.

Explanation:

When a stretch string fixed at both ends is set into vibration, it produces its lowest sound of possible note called the fundamental frequency.  Under certain conditions on the string, higher frequencies called harmonics or overtones can be produced.

The frequency of the forth harmonic is the third overtone of the string and can be determined by:

          f = [tex]\frac{2}{L}[/tex][tex]\sqrt{\frac{T}{m} }[/tex]

Given that; L = 48.0 cm = 0.48 m,

                 m = 0.3 g = 0.0003 Kg,

                 T = 4.0 N,

         f = [tex]\frac{2}{0.48}[/tex][tex]\sqrt{\frac{4}{0.0003} }[/tex]

         f = 4.1667 × 115.4701

           = 481.1252

        f = 481.13 Hz

The frequency of the 4th harmonic of the string is 481.13 Hz.

a girl pushes an 18.15 kg wagon with a force of 3.63 N. what is the acceleration?

A. 0.06 m/s2
B. 5 m/s2
C. 0.2 m/s2

Answers

Answer:0.2 m/s^2

Explanation:

mass=18.15kg

Force=3.63N

Acceleration=force ➗ mass

Acceleration=3.63 ➗ 18.15

Acceleration=0.2 m/s^2

A girl pushes a wagon of mass 18.15 kg with a force of 3.63 N, so the acceleration of the wagon will be 0.2 m/s².

What is acceleration?

In mechanics, acceleration is the measure of how rapidly an object's velocity changes over time. Accelerations as a vector quantity. An object's acceleration depends on the direction of the net force exerted on it.

A vector quantity, acceleration, is something that has both a magnitude and a direction. As a vector quantity, velocity is also. The ratio of the velocity vector change over a time interval to that interval is the definition of acceleration.

Mass, m =18.15 kg

Force, f = 3.63 N

Force = m × a

a= f / m

a =3.63 / 18.15

a = 0.2 m/s²

Hence, the acceleration of the wagon will be 0.2 m/s².

To get more information about acceleration :

https://brainly.com/question/12550364

#SPJ2

In this circuit the battery provides 3 V, the resistance R1 is 7 Ω, and R2 is 5 Ω. What is the current through resistor R2? Give your answer in units of Amps. An Amp is 1 Coulomb of charge flowing through a cross-sectional area of the wire per second - that's a lot of charge per second and will warm up a typical wire quite a bit! Most devices have circuits with larger resistors - kLaTeX: \OmegaΩ (103 LaTeX: \OmegaΩ) and MLaTeX: \OmegaΩ (106 LaTeX: \OmegaΩ) are common.

Answers

Answer:

The current pass the [tex]R_2[/tex] is  [tex]I = 0.25 A[/tex]

Explanation:

The diagram for this question is shown on the first uploaded image  

From the question we are told that

    The voltage  is  [tex]V = 3V[/tex]

     The first resistance is  [tex]R_1 = 7 \Omega[/tex]

     The second resistance is  [tex]R_2 = 5 \Omega[/tex]

Since the resistors are connected in series their equivalent resistance is  

       [tex]R_{eq} = R_1 +R_2[/tex]

Substituting values

         [tex]R_{eq} = 7 + 5[/tex]

         [tex]R_{eq} = 12 \Omega[/tex]

Since the resistance are connected in serie the current passing through the circuit  is the same current passing through [tex]R_2[/tex] which is mathematically evaluated as

        [tex]I = \frac{V}{R_{eq}}[/tex]

Substituting values  

      [tex]I = \frac{3}{12}[/tex]

      [tex]I = 0.25 A[/tex]

An athlete is working out in the weight room. He steadily holds 50 kilograms above his head for 10 seconds. Which statement is true about this situation?

Answers

Answer:

Answer: the true statement form the given statements is “the athletes is not doing any work because he does not move weight”

Explanation:

The athlete isn’t doing any work because he doesn’t move the weight is the correct statement.

What is Work? Work is the energy transferred to or from an object via the application of force along a displacement.Work = Force x Displacement.

How to solve this Problem?The weight of an object given is 50kgsThe time of holding an object given is 10 secondsWe need to justify the statements

Here ,

There is no displacement that means displacement is zero.If displacement is zero then work done will also be zero

Hence there is no work done by the athlete

Therefore ,The athlete isn’t doing any work because he doesn’t move the weight is the correct statement

Learn more about Work done here

https://brainly.com/question/25573309

#SPJ2

(20) A rocket is launched vertically. At time t = 0 seconds, the rocket’s engine shuts down. At the time, the rocket has reached an altitude of 500m and is rising at a velocity of 125 m/s. Gravity then takes over. The height of the rocket as a function of time is h(t)=-9.8/2 t^2+125t+500,t>0. Using your function file from HW2A: Generate a plot of height (vertical axis) vs. time (horizontal axis) from 0 to 30 seconds. Include proper axis labels. Find the maximum height and the time at which it occurs: Analytically, showing your steps and equations. This part should be done entirely in the write-up: no coding Using the data cursor on the plot. Using the MAX function on your data from part (a) Using FMINSEARCH on your m file Comment on the differences between the methods. How closely does each method match the "true" (analytical) value? Find the time when the rocket hits the ground: Analytically, showing your equations. This part should be done entirely in the write-up: no coding Using the data cursor on the plot. Using FZERO on your m file Comment on the differences between the methods in each of part (B) and (C). How closely does each method match the "true" (analytical) value? Use a quantitative comparison to make your argument.

Answers

Answer:

Explanation:

Given that,

h(t) = -9.8t² / 2 + 125t + 500

for t > 0.

At t = 0, the rocket is at height h = 500m, at a velocity of Vo = 125m/s.

We want to find the maximum height reached by rocket

Using mathematics maxima and minima

let find the turning point when dh/dt = 0

dh/dt = -9.8t + 125

dh / dt = 0 = -9.8t + 125

9.8t = 125

t = 125 / 9.8

t = 12.76s

Let find the turning point to know if this time t = 12.76 is maximum or minimum point

Let find d²h / dt²

d²h / dt² = -9.8

Since, d²h/dt² < 0, then, at t = 12.76s is the maximum points.

Then, the maximum height reached is

h =  -9.8t² / 2 + 125t + 500

h =  -9.8(12.76)² / 2 + 125(12.76) + 500

h = -797.80 + 1595 + 500

h = 1297.2 m

The maximum height reached is 1297.2 m

From the attachment, the maximum height is 1297.2m at t = 12.76sec.

Comment, the result are the same for both the analysis aspect and the graphical aspect.

During the process of mountain building, earthquakes sometimes occur along continental-continental convergent boundaries. Which statement best describes the motion of the plates along these boundaries that causes the earthquakes?

The plates push apart from each other with no subduction.
Subduction occurs with the less-dense plate sinking below the other plate.
Subduction occurs with the more-dense plate sinking below the other plate.
The plates smash together with no subduction.

Answers

Answer:

The correct answer is The plates smash together with no subduction. I just took this on Edge. Glad I could help!

The statement best describes the motion of the plates along these boundaries that causes the earthquakes is that The plates smash together with no subduction

For better understanding let's explain what the answer means

it is said that the meeting point of two tectonic plates will causea convergent plate boundary to be formed. one of the converging plates will move under the other as it is known as subduction

From the above we can therefore say that the answer The statement best describes the motion of the plates along these boundaries that causes the earthquakes is that The plates smash together with no subduction, is correct

Learn more motion of the plates along these boundaries from:

https://brainly.com/question/23731885

Which of the following BEST summarizes the relationship between groups and culture and critical thinking?

Answers

Answer:

Groups and culture helps in influencing our values,ethics and beliefs. This influence should always be questioned through the process of thinking critically.

This best summarizes the relationship between groups and culture and critical thinking.

A pipe branches symmetrically into two legs of length L, and the whole system rotates with angular speed ω around its axis of symmetry. Each branch is inclined at angle α to the axis of rotation. Liquid enters the pipe steadily, with zero angular momentum, at volume flow rate Q. The pipe diameter, D, is much smaller than L. Obtain an expression for the external torque required to turn the pipe. What additional torque would be required to impart angular acceleration ω_ ?

Answers

Answer:

Check the explanation

Explanation:

Kindly check the attached images below to see the step by step explanation to the question above.

"It is impossible to devise a process which may convert heat, extracted from a single
reservoir, entirely into work without leaving any change in the working system”.
Use the Second Law of Thermodynamics to state our inability to utilize the heat contents
of oceans and atmosphere which contains a large amount of heat energy but cannot be
converted into useful mechanical work You may use the concept of heat engine to discuss
this​

Answers

Answer:

According to the second law of thermodynamics, we are unable to use the heat of the ocean and the atmosphere because we do not have a reservoir that has a temperature lower than the ocean or the atmosphere.

Explanation:

As you already know, the ocean and atmosphere have a lot of thermal energy, however, we are unable to convert this energy into mechanical energy that would be useful for our activities. This can be explained by the second law of thermodynamics, since it states that the presence of two bodies with different temperatures is necessary for it to be possible to transform heat into work.

In this case, to transform the thermal energy of the ocean and the atmosphere into mechanical energy we would need the existence of a thermal motor, which is only possible to be established when there is a body with high thermal energy and a sink, a reservoir, with low thermal energy, which will be the place where the heat will be expelled, to be converted into work. We do not have a reservoir with less thermal energy than the ocean and the atmosphere, so we cannot use their energy.

A particular coil has 100 turns and a diameter of 6.0 m. When it's time for a measurement, a 4.5 A current is turned on. The large diameter of the coil means that the field in the water flowing directly above the center of the coil is approximately equal to the field in the center of the coil. The field is directed downward and the water is flowing east. The water is flowing above the center of the coil at 1.5 m/s .

What is the magnitude of the field at the center of the coil?

Answers

Answer:

The magnetic field at the center of the coil = 5.23 * 10 ^ -5 T

Explanation:

Information from the question:

Number of turns of the coil = 100 turns

The diameter of the coil = 6 m

The radius of the coil = diameter / 2 = 3 m

The coil current = 2.5 A

Formula : The Magnetic field at the center of the coil =

                                  k * number of turns * current / 2 * radius

Therefore, The Magnetic field at the center of the coil=

                                 (4 * [tex]\pi[/tex] * 10 ^ -7 * 100 * 2.5 ) / (2 * 3)

The Magnetic field at the center of the coil = 5.23 * 10 ^ -5 T

plzzz help will mark the brainliest

Answers

Ciara is winging....etc
The answer is : 0.60 N, toward the center of the circle


A satellite....etc
The Answer is : 7400 m/s


What is the .....etc
The Answer is : 2.60 m/s

A brick is dropped from a high scaffold. a. What is its velocity after 4.0s ?

b. How far does the brick fall during this time ?

Answers

Answer:

A: 1.962

B: 3.924

Explanation:

g = G *M /R^2

g = 9.807*M/R^2 the gravitational constant of ground level on earth is about 9.807

g = 9.807*5lbs/R^2 the average brick is about 5 pounds.

g = 9.807*5*10^2.   I'm assuming the height is around ten feet to help you out.

with these numbers plugged in you get an acceleration of 0.4905 a final velocity after 4 seconds 1.962. It's height fallen after 4 seconds is 3.924.

( M = whatever the brick weighs it's not specified in the question)

(R = the distance from the ground or how high the scaffold is)

(hopefully you can just plug your numbers in there hope this helps)

What spectacles are required for reading purposes by a person whose near point is 2.0m

Answers

Answer:Convex lens spectacles is required for reading purpose..

Explanation:

I don't say you have to mark my ans as brainliest but if it has really helped you please don't forget to thank me...


What is an independent variable?
A. A variable that is intentionally changed during an experiment
B. A variable that depends on the experimental variable
C. A variable that is not used in an experiment
D. A variable that is unknown during the experiment

Answers

Answer:

The answer is A

Explanation:

Independent variables don't have to depend on other factors of the experiment because they're independent

Answer:

A.

Explanation:

Independent variables don't have to depend on other factors of the experiment because they're independent.

Is mercury (the planet) rocky or gaseous(meaning relating to or having the characteristics of a gas.)

Answers

Answer:

Mercury is rocky

Explanation:

Answer:

Rocky

Explanation:

It has no atmosphere so it cannot hold gas.

The universal law of gravitation states that the force of attraction between two objects depends on which quantities?
the masses of the objects and their densities
the distance between the objects and their shapes
the densities of the objects and their shapes
the masses of the objects and the distance between them
Save and Exit
Next
Subm
Kandretum

Answers

Answer:depends on the masses of the objects and the distance between them

Explanation:

According to Newton's law of universal gravitation,the force of attraction between two objects depends on the masses of the objects and the distance between them

Question 7
Review
Which particles are not affected by the strong force?
A.
hadrons
B.
protons
C.
neutrons
D
electrons
Submit A
Hide Toolbar

Answers

Answer:

Electron

Explanation:

Because electron are not hadrons so electron are not affected by strong force

Particles that can not be affected by strong forces are electrons.

What are electrons?

Electrons are the rotating material around the nucleus of an atomic element in orbit.

Atoms have electrostatic energy between their electrons. This force is not broken by a force as strong as nuclear power.

What are strong forces?

Strong force is a fundamental interaction of nature that acts between subatomic particles of matter.

There are four basic forces in nature:

Gravity: the gravitational force used between any heavy objects. It has an infinite range.Electrical energy: energy used between electrically charged objects. It can be either attraction or repulsion.Nuclear power: is the magnetic field that responds to the binding of protons and neutrons within the nucleus of an atom. It only works for very short distances.Weak nuclear power: a force that causes nuclear decay. It only works for very short distances.

Therefore, particles that are unaffected by strong force are electrons.

To learn more about strong force here

https://brainly.com/question/9910823

#SPJ2

I need some help!!!!!!!!!

Answers

Answer:

The Object will immediately begin moving toward the left

Explanation:

Because the force of thirteen is greater than ten and applied to the opposite side

A cobalt-60 source with activity 2.60×10-4 Ci is embedded in a tumor that has
mas 0.20 kg. The source emits gamma photons with average energy 1.25 MeV.
Half the photons are absorbed in the tumor, and half escape.
i. What energy is delivered to the tumor per second? [4 marks]
ii. What absorbed dose, in rad, is delivered per second? [2 marks]
iii. What equivalent dose, in rem, is delivered per second if the RBE for
these gamma rays is 0.70? [2 marks]
Page 6 of 7
iv. What exposure time is required for an equivalent dose of 200 rem? [2
marks]
B. A laser with power output of 2.0 mW at a wavelength of 400 nm is projected
onto a Calcium metal. The binding energy is 2.31 eV.
i. How many electrons per second are ejected? [6 marks]
ii. What power is carried away by the electrons? [4 marks]
C. A hypodermic needle of diameter 1.19 mm and length 50 mm is used to
withdraw blood from a patient? How long would it take for 500 ml of blood to be
taken? Assume a blood viscosity of 0.0027 Pa.s and a pressure in the vein of
1,900 Pa. [10 marks]
D. A person with lymphoma receives a dose of 35 gray in the form of gamma
radiation during a course of radiotherapy. Most of this dose is absorbed in 18
grams of cancerous lymphatic tissue.
i. How much energy is absorbed by the cancerous tissue? [2 marks]
ii. If this treatment consists of five 15-minute sessions per week over the
course of 5 weeks and just one percent of the gamma photons in the
gamma ray beam are absorbed, what is the power of the gamma ray
beam? [4 marks]
iii. If the gamma ray beam consists of just 0.5 percent of the photons
emitted by the gamma source, each of which has an energy of 0.03
MeV, what is the activity, in Curies, of the gamma ray source? [4 marks]
E. A water heater that is connected across the terminals of a 15.0 V power supply
is able to heat 250 ml of water from room temperature of 25°C to boiling point
in 45.0 secs. What is the resistance of the heater? The density of water is 1,000
kg/m2 and the specific heat capacity of water is 4,200 J/kg/°C. [10 marks]

Answers

Answer:

A i. E = 9.62 × 10⁻⁷ J/s

ii. The absorbed dose is 4.81 × 10⁻⁶ Gy

iii. The equivalent dose is  3.37 × 10⁻⁴ rem/s

iv.  t = 593471.81 seconds

B. i. 4.025 × 10¹⁵/s

ii. 0.512 mW

C. 7218092.2 seconds

D. i. 6.3 × 10⁻¹ J

ii. 1.4 × 10⁻² W

iii. 1.57 × 10³ Curie

E. 0.129 Ω

Explanation:

The given parameters are;

Mass of tumor = 0.20 kg

Activity of Cobalt-60 = 2.60 × 10⁻⁴ Ci

Photon energy = 1.25 MeV

(i) The energy, E, delivered to the tumor is given by the relation;

[tex]E = \frac{1}{2}\left (Number \, of \, decay / seconds \right )\times \left (Energy \, of \, photon \right )[/tex]

[tex]E = \frac{1}{2}\left (2.6\times 10^{-4}Ci )\times \left (\frac{3.70\times 10^{10}decays/s}{1 Ci} \right )\times 1.25\times 10^{6}eV\times \frac{1.6\times 10^{-19}J}{1eV}[/tex]

E = 9.62 × 10⁻⁷ J/s

(ii) The equation for absorbed dose is given as follows;

Absorbed dose, D, in Grays Gy = (Energy Absorbed Joules J)/Mass kg

Therefore, absorbed dose = (9.62 × 10⁻⁷ J/s)/( kg) = 4.81 × 10⁻⁶ Gy

1 Gray = 100 rad

4.81 × 10⁻⁷ Gy = 100 × 4.81 × 10⁻⁶ = 4.81 × 10⁻⁴ rad/s

(iii) Equivalent dose, H, is  given by the relation;

H = D × Radiation factor, [tex]w_R[/tex]

∴ H = 0.7 × 4.81 × 10⁻⁴ rad/s = 3.37 × 10⁻⁴ Sv = 3.37 × 10⁻⁴ rem/s

(iv) The exposure time required for an equivalent dose of 200 rem is given as follows;

[tex]\dot{H} = \dfrac{H}{t}[/tex]

Therefore;

[tex]t= \dfrac{200}{{3.37 \times 10^{-4}} } = 593471.81 \, s[/tex]

∴ t = 6.9 days

B. The number of electrons ejected is given by the relation;

[tex]N = \frac{P}{E} = \frac{P \times \lambda}{hc}[/tex]

[tex]N = \dfrac{2.0 \times 10^{-3} \times 400 \times 10^{-9}}{6.626 \times 10^{-34} \times 3 \times 10^8} = 4.025 \times 10^{15}/s[/tex]

(ii) The power carried by the electron

The energy carried away by the electrons is given by the relation;

[tex]KE_e = hv - \Phi[/tex]

[tex]KE_e = \frac{6.626 \times 10^{-34} \times 3 \times 10^8}{400 \times 10^{-9}} - 2.31 \times \frac{1.6 \times 10 ^{-19} }{1}[/tex]

[tex]KE_e = 4.9695 \times 10^{-19} - 3.696 \times 10 ^{-19} = 1.2735 \times 10^{-19} J[/tex]

Power, P[tex]_e[/tex], carried away by the electron = 4.025 × 10¹⁵ × 1.2735 × 10⁻¹⁹ = 0.512 mW

C. The given parameters are;

d = 1.19 mm, ∴ r = 1.19/2 = 0.595 × 10⁻³ m

l = 50 mm = 5 × 10⁻³ m

V = 500 ml = 5 × 10⁻⁴ m³

η = 0.0027 Pa

p = 1,900 Pa.

[tex]\dfrac{V}{t} = \dfrac{\pi }{8} \times \dfrac{P/l}{\eta } \times r^4[/tex]

[tex]t = \dfrac{8\times \eta\times V\times l }{\pi \times P \times r^4}[/tex]

[tex]t = \dfrac{8\times 0.0027 \times 5 \times 10^{-4} \times 5 \times 10^{-2} }{\pi \times 1900 \times (0.595 \times 10^{-4} )^4}[/tex]

t = 7218092.2 seconds

D) i. Energy absorbed is given by the relation;

E = m×D

Where:

D = 35 Gray = 35 J/kg

m = 18 g = 18 × 10⁻³ kg

∴ E = 35 × 18 × 10⁻³ = 6.3 × 10⁻¹ J

ii. Total time for treatment = 15 × 5 = 75 minutes

Energy absorbed = 6.3 × 10⁻¹ × 100 = 63 J

Power = Energy(in Joules)/Time (in seconds)

∴ Power = 63/(75×60) = 1.4 × 10⁻² W

iii. Whereby the power is provided by 0.5% of the photons emitted by the source, we have;

[tex]P_{source}= \frac{P_{beam}}{0.005} =\frac{0.0014}{0.005} =0.28 \, W[/tex]

1 MeV = 1.60218 × 10⁻¹³ J

0.03 MeV = 0.03 × 1.60218 × 10⁻¹³ J = 4.80654 × 10⁻¹⁵ J/photon

Therefore, the number of disintegration per second = 0.28 J/s ÷  4.80654 × 10⁻¹⁵ J/photon = 5.83 × 10¹³ disintegrations per second

1 Curie = 3.7 × 10¹⁰  disintegrations per second

Hence, 5.83 × 10¹³ disintegrations per second = (5.83 × 10¹³)/(3.7 × 10¹⁰) Curie

= 1.57 × 10³ Curie

E. The parameters given are;

Density of water = 1000 kg/m³

Volume of water = 250 ml = 0.00025 m³

Initial temperature, T₁, = 25°C

Final temperature, T₂, = 100°C

Change in temperature, ΔT = 100 - 25 = 75°

Specific heat capacity of the water = 4200 J/kg/°C

Mass of water = Density × Volume = 1000 × 0.00025 = 0.25 kg

∴ Heat supplied = 4200 × 0.25 × 75 = 78,750 J

Time to heat the water = 45.0 sec

Therefore, power = Energy/time = 78750/45 = 1750 W

The formula for electrical power = I²R =VI = V²/R

Therefore, where V = 15.0 V, we have;

15²/R = 1750

R = 15²/1750 = 0.129 Ω.

The resistance of the heater = 0.129 Ω.

Two wires, both with current out of the page, are next to one another. The wire on the left has a current of 1 A and the wire on the right has a current of 2 A. We can say that:

A. The left wire attracts the right wire and exerts twice the force as the right wire does.
B. The left wire attracts the right wire and exerts half the force as the right wire does.
C. The left wire attracts the right wire and exerts as much force as the right wire does.
D. The left wire repels the right wire and exerts twice the force as the right wire does.
E. The left wire repels the right wire and exerts half the force as the right wire does.
F. The left wire repels the right wire and exerts as much force as the right wire does.

Answers

Answer:

C. The left wire attracts the right wire and exerts as much force as the right wire does.

Explanation:

To know what is the answer you first take into account the magnetic field generated by each current, for a distance of d:

[tex]B_1=\frac{\mu_oI_1}{2\pi d}=\frac{\mu_o}{2\pi d}(1A)\\\\B_2=\frac{\mu_oI_2}{2\pi d}=\frac{\mu_o}{2\pi d}(2A)=2B_1\\\\[/tex]

Next, you use the formula for the magnetic force produced by the wires:

[tex]\vec{F_B}=I\vec{L}\ X \vec{B}[/tex]

if the direction of the L vector is in +k direction, the first wire produced a magnetic field with direction +y, that is, +j and the second wire produced magnetic field with direction -y, that is, -j (this because the direction of the magnetic field is obtained by suing the right hand rule). Hence, the direction of the magnetic force on each wire, produced by the other one is:

[tex]\vec{F_{B1}}=I_1L\hat{k}\ X\ B_2(-\hat{j})=I_1LB_2\hat{i}=(2A^2)\frac{L\mu_o}{2\pi d}\hat{i}\\\\\vec{F_{B2}}=I_2L\hat{k}\ X\ B_2(\hat{j})=I_2LB_1\hat{i}=-(2A^2)\frac{L\mu_o}{2\pi d}\hat{i}[/tex]

Hence, due to this result you have that:

C. The left wire attracts the right wire and exerts as much force as the right wire does.

This is a measure of quantity of matter

Answers

Answer:

Mass

Explanation:

Mass is the measure of amount of matter contained within any substance and hence mass determines the weight. Unit of mass is kilogram as per ISI system of units.  

Mass is measured through a balance. The more is the mass of an object, the more the balance tilts towards the object side.  

Weight is equal to product of mass and the gravitational constant i.e 9.8m/s^2

Other Questions
What causes a wage-price spiral, and how does it affect the economy? The measures of the angles of a triangle are 25 degrees, 87 degrees, and 68 degrees. Is the triangle acute, right, or obtuse? hich of the following organisms receives at least some energy from plants? A. coyotes B. rabbits C. deer D. all of these Chargaffs rule states Read the excerpt from "Let Bindi Have the Limelight." As one important being left Bindi's life, another entered it. He certainly wasn't larger than life like her dad had been. In fact, he wasn't very large at all. That's because Candy . . . was a rat. What does the hyperbole help you understand?A. how happy Bindi was to receive a new petB. how important Bindis dad was to herC. how big the personality of Bindis dad wasD. how small Bindis pet rat is What is the volume of the prism below?O A. 144 units 3B. 56 unitsC. 288 units 3D. 180 units 3 The musical form that repeats specific 12 bar harmonic pattern is? Only today I wish I didn't have only eleven years rattling inside me like pennies in a tin Band-Aid box. Today I wish I was one hundred and two instead of eleven because if I was one hundred and two, I'd have known what to say when Mrs. Price put the red sweater on my desk. I would've known how to tell her it wasn't mine instead of just sitting there with that look on my face and nothing coming out of my mouth.Eleven,Sandra CisnerosRead the passage. In a paragraph, describe the conflict and explain what it tells you about Rachel. Consider the following estimates from the early 2010s of shares of income to each group. Country Poorest 40% Next 30% Richest 30% Bolivia 10 25 65 Chile 10 20 70 Uruguay 20 30 50 1.) Using the 4-point curved line drawing tool, plot the Lorenz curve for Bolivia. Properly label your curve. 2.) Using the 4-point curved line drawing tool, plot the Lorenz curve for Uruguay. Properly label your curve. Carefully follow the instructions above, and only draw the required objects. Which country has the most nearly equal income distribution? Chile Uruguay Bolivia . The two-way table represents the results of a random survey taken to determine the preferred vehicle for male and female drivers. Given that the participant is a female, which choice is the conditional relative frequency that she prefers an SUV? Which unit would be best for measuring the thickness of a coin?kilometer()) centimetermeter)) milli When Jackie Robinson broke the color line in professional baseball,he was welcomed enthusiastically nearly everywhere.he received threats, abuse, and harsh treatment, even from other teams.other players and fans barely noticed. Can you guys answer this question please ASAP, thanks will mark brainliest volume .................... What are the restrictions placed upon congress power to legislate? How to change miles per hour to feet per second? 100 POINTS!!! NO JOKE!!! (IF YOU ANSWER THE QUESTIONS) It's a little time consuming but I want to see who can answer the fastest. I would really appreciate the answers to these asap. ONLY ANSWER IF YOU KNOW HOW TO DO THESE!!! They're easy but I don't want to do them. Appreciate = Will mark Brainliest if correctThanks so much If there are 7 people in a family, 4 grandparents, a grandchild a mom and dad, what is the average age of each person, if all together they are 381 years old? Decide if the clause in capital letters is DEPENDENT or INDEPENDENT.We all need heroes WHO SERVE AS INSPIRATION.A. A dependent clauseB. An independent clause FIRST PERSON TO ANSWER GETS BRANLYSEST After Hunter drank a glass of milk, sugar levels in his blood increased. This stimulated the hormone insulin to be released. Insulin helped the sugar move from his blood into cells, so his blood sugar levels decreased. Which statement explains what happened next? Positive feedback caused his pancreas to stop secreting insulin. Negative feedback caused his pancreas to stop secreting insulin. Negative feedback caused his pancreas to produce more insulin. Positive feedback caused his pancreas to produce more insulin.